Comparing $ln 1000$, $sqrt[5]{1000}$, $3^{1000}$, and $1000^{15}$ without calculator












2












$begingroup$


In my Pre-Calculus class we were given the following problem:




Put the following four values in order from smallest to largest: $ln 1000$, principal $5$th root of $1000$, $3^{1000}$, and $1000^{15}$.




Using a calculator, I was able to figure out that the correct order is as follows: principal $5$th root of $1000, ln 1000, 1000^{15}, 3^{1000}$.



However, we are supposed to be able to solve this problem without a calculator, and then it becomes much more difficult for me, to say the least.



I know that any exponential growth function (e.g., $3^x$) "eventually" outpaces any polynomial function (e.g., $x^{15}$).



So I suppose the teacher could just be assuming that we will see $x = 1000$ and conclude that $1000$ seems "sufficiently large" for the exponential, $3^{1000}$, to have outpaced $1000^{15}$. Nonetheless, that is somewhat unsatisfying.



Furthermore, it still provides no answer to the even trickier question of how you know $ln 1000$ is greater than the principal $5$th root of $1000$. This becomes clear when you graph on a calculator, but as I said I'm supposed to be able to solve this problem without a calculator. And, indeed, the difference between $ln 1000$ and the principal $5$th root of $1000$ is only about $3$, according to my calculator -- so which of the two is bigger seems hardly obvious to me from a quick glance at the two quantities (without plugging them into a calculator).



It's an incredibly fascinating problem, but I just can't figure it out. Any answers would be greatly appreciated. Thank you so much!! :)










share|cite|improve this question











$endgroup$








  • 1




    $begingroup$
    Welcome to MathSE. Please read this tutorial on how to typeset mathematics on this site.
    $endgroup$
    – N. F. Taussig
    Dec 2 '18 at 12:26
















2












$begingroup$


In my Pre-Calculus class we were given the following problem:




Put the following four values in order from smallest to largest: $ln 1000$, principal $5$th root of $1000$, $3^{1000}$, and $1000^{15}$.




Using a calculator, I was able to figure out that the correct order is as follows: principal $5$th root of $1000, ln 1000, 1000^{15}, 3^{1000}$.



However, we are supposed to be able to solve this problem without a calculator, and then it becomes much more difficult for me, to say the least.



I know that any exponential growth function (e.g., $3^x$) "eventually" outpaces any polynomial function (e.g., $x^{15}$).



So I suppose the teacher could just be assuming that we will see $x = 1000$ and conclude that $1000$ seems "sufficiently large" for the exponential, $3^{1000}$, to have outpaced $1000^{15}$. Nonetheless, that is somewhat unsatisfying.



Furthermore, it still provides no answer to the even trickier question of how you know $ln 1000$ is greater than the principal $5$th root of $1000$. This becomes clear when you graph on a calculator, but as I said I'm supposed to be able to solve this problem without a calculator. And, indeed, the difference between $ln 1000$ and the principal $5$th root of $1000$ is only about $3$, according to my calculator -- so which of the two is bigger seems hardly obvious to me from a quick glance at the two quantities (without plugging them into a calculator).



It's an incredibly fascinating problem, but I just can't figure it out. Any answers would be greatly appreciated. Thank you so much!! :)










share|cite|improve this question











$endgroup$








  • 1




    $begingroup$
    Welcome to MathSE. Please read this tutorial on how to typeset mathematics on this site.
    $endgroup$
    – N. F. Taussig
    Dec 2 '18 at 12:26














2












2








2


1



$begingroup$


In my Pre-Calculus class we were given the following problem:




Put the following four values in order from smallest to largest: $ln 1000$, principal $5$th root of $1000$, $3^{1000}$, and $1000^{15}$.




Using a calculator, I was able to figure out that the correct order is as follows: principal $5$th root of $1000, ln 1000, 1000^{15}, 3^{1000}$.



However, we are supposed to be able to solve this problem without a calculator, and then it becomes much more difficult for me, to say the least.



I know that any exponential growth function (e.g., $3^x$) "eventually" outpaces any polynomial function (e.g., $x^{15}$).



So I suppose the teacher could just be assuming that we will see $x = 1000$ and conclude that $1000$ seems "sufficiently large" for the exponential, $3^{1000}$, to have outpaced $1000^{15}$. Nonetheless, that is somewhat unsatisfying.



Furthermore, it still provides no answer to the even trickier question of how you know $ln 1000$ is greater than the principal $5$th root of $1000$. This becomes clear when you graph on a calculator, but as I said I'm supposed to be able to solve this problem without a calculator. And, indeed, the difference between $ln 1000$ and the principal $5$th root of $1000$ is only about $3$, according to my calculator -- so which of the two is bigger seems hardly obvious to me from a quick glance at the two quantities (without plugging them into a calculator).



It's an incredibly fascinating problem, but I just can't figure it out. Any answers would be greatly appreciated. Thank you so much!! :)










share|cite|improve this question











$endgroup$




In my Pre-Calculus class we were given the following problem:




Put the following four values in order from smallest to largest: $ln 1000$, principal $5$th root of $1000$, $3^{1000}$, and $1000^{15}$.




Using a calculator, I was able to figure out that the correct order is as follows: principal $5$th root of $1000, ln 1000, 1000^{15}, 3^{1000}$.



However, we are supposed to be able to solve this problem without a calculator, and then it becomes much more difficult for me, to say the least.



I know that any exponential growth function (e.g., $3^x$) "eventually" outpaces any polynomial function (e.g., $x^{15}$).



So I suppose the teacher could just be assuming that we will see $x = 1000$ and conclude that $1000$ seems "sufficiently large" for the exponential, $3^{1000}$, to have outpaced $1000^{15}$. Nonetheless, that is somewhat unsatisfying.



Furthermore, it still provides no answer to the even trickier question of how you know $ln 1000$ is greater than the principal $5$th root of $1000$. This becomes clear when you graph on a calculator, but as I said I'm supposed to be able to solve this problem without a calculator. And, indeed, the difference between $ln 1000$ and the principal $5$th root of $1000$ is only about $3$, according to my calculator -- so which of the two is bigger seems hardly obvious to me from a quick glance at the two quantities (without plugging them into a calculator).



It's an incredibly fascinating problem, but I just can't figure it out. Any answers would be greatly appreciated. Thank you so much!! :)







algebra-precalculus logarithms exponentiation radicals number-comparison






share|cite|improve this question















share|cite|improve this question













share|cite|improve this question




share|cite|improve this question








edited Dec 2 '18 at 14:08









user21820

38.8k543153




38.8k543153










asked Dec 2 '18 at 5:37









Will Will

113




113








  • 1




    $begingroup$
    Welcome to MathSE. Please read this tutorial on how to typeset mathematics on this site.
    $endgroup$
    – N. F. Taussig
    Dec 2 '18 at 12:26














  • 1




    $begingroup$
    Welcome to MathSE. Please read this tutorial on how to typeset mathematics on this site.
    $endgroup$
    – N. F. Taussig
    Dec 2 '18 at 12:26








1




1




$begingroup$
Welcome to MathSE. Please read this tutorial on how to typeset mathematics on this site.
$endgroup$
– N. F. Taussig
Dec 2 '18 at 12:26




$begingroup$
Welcome to MathSE. Please read this tutorial on how to typeset mathematics on this site.
$endgroup$
– N. F. Taussig
Dec 2 '18 at 12:26










4 Answers
4






active

oldest

votes


















5












$begingroup$

Without a calculator you may proceed as follows:




  • $4^5 = 1024 Rightarrow sqrt[5]{1000} < 4$

  • $2 < e < 3 mbox{ and } 2^{10} > 1000 = e^{ln{1000}} > 3^6 Rightarrow 6 < ln 1000 < 10$

  • $3^{1000} = left(3^{frac{1000}{15}}right)^{15} > left(3^{60}right)^{15} > 1000^{15}$






share|cite|improve this answer









$endgroup$









  • 2




    $begingroup$
    Or for the last without fractions in the exponent, $3^{1000}=(3^{20})^{50}>10^{50}>10^{45}=1000^{15}$
    $endgroup$
    – LutzL
    Dec 2 '18 at 10:49



















3












$begingroup$

$
ln(1000)= 3ln(10)
$
is between 6 and 9 because $2<ln(10)<3$



this is clearly bigger than $1000^{0.2}$ because $$ (ln(1000))^5 > 6^5>4^5=1024>1000$$






share|cite|improve this answer









$endgroup$





















    2












    $begingroup$

    You can use a change of base on the exponentials to see which should be larger:



    $$
    a^b = c^{blog_c(a)}
    $$



    We can use base 10 or, as I prefer, the natural log:



    $$
    3^{1000} = e^{1000ln(3)} approx e^{1099}
    $$



    and



    $$
    1000^{15} = e^{15ln(1000)} approx e^{104}
    $$



    It should be clear that the $5^text{th}$ root of $1000$ will be smaller, but you can proceed the same:



    $$
    sqrt[5]{1000} = 1000^frac{1}{5} = e^{frac{1}{5}ln(1000)} approx e^{1.4}
    $$



    EDIT:



    I just realized you were supposed to not use a calculator. Because of this, the base $10$ logarithm makes more sense:



    begin{align*}
    3^{1000} &= 10^{1000*log_{10}(3)} \
    1000^{15} &= 10^{15*log_{10}(1000)} = 10^{45} &text{$log_{10}left(10^3right) = 3$}\
    1000^{frac{1}{5}} &= 10^{frac{1}{5}log_{10}(1000)} = 10^{frac{3}{5}} &text{$log_{10}left(10^3right) = 3$}
    end{align*}



    All you need to do is show that $1000log_{10}(3) > 45$. We can approximate $log_{10}(3)$. We know that $3^2 < 10 < 3^3$, therefore $10^frac{1}{3} < 3 < 10^frac{1}{2}$ and thus $frac{1}{3} < log_{10}(3) < frac{1}{2}$. We use the smaller number (we want to "over" correct, by going as small as possible) and find that:



    $$
    frac{1000}{3} > 45
    $$



    Second Edit : $ln(1000)$



    Again, let's switch to base 10:



    $$
    log_a(x) = frac{log_b(x)}{log_b(a)}
    $$



    So we have:



    $$
    ln(1000) = frac{log_{10}(1000)}{log_{10}(e)} = frac{3}{log_{10}(e)}
    $$



    We can approximate $log_{10}(e)$ in the same way we approximated $log_{10}(3)$. We know that $2 < e < 3$. $2^3 < 10 < 2^4$ and (again) $3^2 < 10 < 3^3$, therefore $frac{1}{4} < log_{10}(2) < frac{1}{3}$ or $frac{1}{3} < log_{10}(3) < frac{1}{2}$. Be careful here, this will tell us that:



    begin{align*}
    frac{3}{frac{1}{2}} < ln(1000) < frac{3}{frac{1}{4}} \
    6 < ln(1000) < 12
    end{align*}



    If you can show that $10^frac{3}{5}$ is less than $6$ or greater than $12$, then you have your answer. So, surely $10^{frac{1}{5}} < 2$ (since $2^5 = 32$) but also $2^3= 8 > 6$ (so that's no good for our range). This is tricky to do, but can be done with some trial and error (hint: $1.5 = frac{3}{2}$ won't work--you'll end up proving that $frac{3}{2} < 10^{1/5}$).






    share|cite|improve this answer











    $endgroup$













    • $begingroup$
      Wait am I supposed to be able to estimate stuff like 1000*ln(3) without a calculator?
      $endgroup$
      – Will
      Dec 2 '18 at 5:47










    • $begingroup$
      @Will Yeah, I just realized that. I edited it. It should make more sense now.
      $endgroup$
      – Jared
      Dec 2 '18 at 5:56










    • $begingroup$
      I will read over it. I'm guessing it will make much more sense. Thank you so much!! One thing: Do you have any idea how to prove that ln 1000 is larger than the root but smaller than either the exponential or the power? You've proven three of the four numbers in the original problem, and I am so thankful. But do you have any thoughts on the other one, ln 1000?
      $endgroup$
      – Will
      Dec 2 '18 at 5:58





















    1












    $begingroup$

    From $$e^2<3^2<10<e^3<3^3$$ we have $$2<2ln 3<ln 10<3<3ln 3 tag 1$$




    • Realize that $sqrt[5]{1000}<ln1000,$ as their fifth powers are in this order: $$1000<2^{10}<6^5<(ln1000)^5$$


    • Now, compute logarithmes of $ln 1000,;1000^{15},;3^{1000}$ and find their estimates with the use of $(1):$



    $$ln3<ln6< color{blue}{ln(ln 1000)}<ln9=2ln3 $$
    $$color{blue}{ln3^{1000}}=1000ln3$$
    $$90ln3<color{blue}{ln 1000^{15}}<135ln3$$
    The numbers are in the same order as their natural logarithmes.



    Putting together both parts, we conclude $$sqrt[5]{1000}<ln 1000<1000^{15}<3^{1000}.$$






    share|cite|improve this answer











    $endgroup$













      Your Answer





      StackExchange.ifUsing("editor", function () {
      return StackExchange.using("mathjaxEditing", function () {
      StackExchange.MarkdownEditor.creationCallbacks.add(function (editor, postfix) {
      StackExchange.mathjaxEditing.prepareWmdForMathJax(editor, postfix, [["$", "$"], ["\\(","\\)"]]);
      });
      });
      }, "mathjax-editing");

      StackExchange.ready(function() {
      var channelOptions = {
      tags: "".split(" "),
      id: "69"
      };
      initTagRenderer("".split(" "), "".split(" "), channelOptions);

      StackExchange.using("externalEditor", function() {
      // Have to fire editor after snippets, if snippets enabled
      if (StackExchange.settings.snippets.snippetsEnabled) {
      StackExchange.using("snippets", function() {
      createEditor();
      });
      }
      else {
      createEditor();
      }
      });

      function createEditor() {
      StackExchange.prepareEditor({
      heartbeatType: 'answer',
      autoActivateHeartbeat: false,
      convertImagesToLinks: true,
      noModals: true,
      showLowRepImageUploadWarning: true,
      reputationToPostImages: 10,
      bindNavPrevention: true,
      postfix: "",
      imageUploader: {
      brandingHtml: "Powered by u003ca class="icon-imgur-white" href="https://imgur.com/"u003eu003c/au003e",
      contentPolicyHtml: "User contributions licensed under u003ca href="https://creativecommons.org/licenses/by-sa/3.0/"u003ecc by-sa 3.0 with attribution requiredu003c/au003e u003ca href="https://stackoverflow.com/legal/content-policy"u003e(content policy)u003c/au003e",
      allowUrls: true
      },
      noCode: true, onDemand: true,
      discardSelector: ".discard-answer"
      ,immediatelyShowMarkdownHelp:true
      });


      }
      });














      draft saved

      draft discarded


















      StackExchange.ready(
      function () {
      StackExchange.openid.initPostLogin('.new-post-login', 'https%3a%2f%2fmath.stackexchange.com%2fquestions%2f3022284%2fcomparing-ln-1000-sqrt51000-31000-and-100015-without-calc%23new-answer', 'question_page');
      }
      );

      Post as a guest















      Required, but never shown

























      4 Answers
      4






      active

      oldest

      votes








      4 Answers
      4






      active

      oldest

      votes









      active

      oldest

      votes






      active

      oldest

      votes









      5












      $begingroup$

      Without a calculator you may proceed as follows:




      • $4^5 = 1024 Rightarrow sqrt[5]{1000} < 4$

      • $2 < e < 3 mbox{ and } 2^{10} > 1000 = e^{ln{1000}} > 3^6 Rightarrow 6 < ln 1000 < 10$

      • $3^{1000} = left(3^{frac{1000}{15}}right)^{15} > left(3^{60}right)^{15} > 1000^{15}$






      share|cite|improve this answer









      $endgroup$









      • 2




        $begingroup$
        Or for the last without fractions in the exponent, $3^{1000}=(3^{20})^{50}>10^{50}>10^{45}=1000^{15}$
        $endgroup$
        – LutzL
        Dec 2 '18 at 10:49
















      5












      $begingroup$

      Without a calculator you may proceed as follows:




      • $4^5 = 1024 Rightarrow sqrt[5]{1000} < 4$

      • $2 < e < 3 mbox{ and } 2^{10} > 1000 = e^{ln{1000}} > 3^6 Rightarrow 6 < ln 1000 < 10$

      • $3^{1000} = left(3^{frac{1000}{15}}right)^{15} > left(3^{60}right)^{15} > 1000^{15}$






      share|cite|improve this answer









      $endgroup$









      • 2




        $begingroup$
        Or for the last without fractions in the exponent, $3^{1000}=(3^{20})^{50}>10^{50}>10^{45}=1000^{15}$
        $endgroup$
        – LutzL
        Dec 2 '18 at 10:49














      5












      5








      5





      $begingroup$

      Without a calculator you may proceed as follows:




      • $4^5 = 1024 Rightarrow sqrt[5]{1000} < 4$

      • $2 < e < 3 mbox{ and } 2^{10} > 1000 = e^{ln{1000}} > 3^6 Rightarrow 6 < ln 1000 < 10$

      • $3^{1000} = left(3^{frac{1000}{15}}right)^{15} > left(3^{60}right)^{15} > 1000^{15}$






      share|cite|improve this answer









      $endgroup$



      Without a calculator you may proceed as follows:




      • $4^5 = 1024 Rightarrow sqrt[5]{1000} < 4$

      • $2 < e < 3 mbox{ and } 2^{10} > 1000 = e^{ln{1000}} > 3^6 Rightarrow 6 < ln 1000 < 10$

      • $3^{1000} = left(3^{frac{1000}{15}}right)^{15} > left(3^{60}right)^{15} > 1000^{15}$







      share|cite|improve this answer












      share|cite|improve this answer



      share|cite|improve this answer










      answered Dec 2 '18 at 6:07









      trancelocationtrancelocation

      9,6851722




      9,6851722








      • 2




        $begingroup$
        Or for the last without fractions in the exponent, $3^{1000}=(3^{20})^{50}>10^{50}>10^{45}=1000^{15}$
        $endgroup$
        – LutzL
        Dec 2 '18 at 10:49














      • 2




        $begingroup$
        Or for the last without fractions in the exponent, $3^{1000}=(3^{20})^{50}>10^{50}>10^{45}=1000^{15}$
        $endgroup$
        – LutzL
        Dec 2 '18 at 10:49








      2




      2




      $begingroup$
      Or for the last without fractions in the exponent, $3^{1000}=(3^{20})^{50}>10^{50}>10^{45}=1000^{15}$
      $endgroup$
      – LutzL
      Dec 2 '18 at 10:49




      $begingroup$
      Or for the last without fractions in the exponent, $3^{1000}=(3^{20})^{50}>10^{50}>10^{45}=1000^{15}$
      $endgroup$
      – LutzL
      Dec 2 '18 at 10:49











      3












      $begingroup$

      $
      ln(1000)= 3ln(10)
      $
      is between 6 and 9 because $2<ln(10)<3$



      this is clearly bigger than $1000^{0.2}$ because $$ (ln(1000))^5 > 6^5>4^5=1024>1000$$






      share|cite|improve this answer









      $endgroup$


















        3












        $begingroup$

        $
        ln(1000)= 3ln(10)
        $
        is between 6 and 9 because $2<ln(10)<3$



        this is clearly bigger than $1000^{0.2}$ because $$ (ln(1000))^5 > 6^5>4^5=1024>1000$$






        share|cite|improve this answer









        $endgroup$
















          3












          3








          3





          $begingroup$

          $
          ln(1000)= 3ln(10)
          $
          is between 6 and 9 because $2<ln(10)<3$



          this is clearly bigger than $1000^{0.2}$ because $$ (ln(1000))^5 > 6^5>4^5=1024>1000$$






          share|cite|improve this answer









          $endgroup$



          $
          ln(1000)= 3ln(10)
          $
          is between 6 and 9 because $2<ln(10)<3$



          this is clearly bigger than $1000^{0.2}$ because $$ (ln(1000))^5 > 6^5>4^5=1024>1000$$







          share|cite|improve this answer












          share|cite|improve this answer



          share|cite|improve this answer










          answered Dec 2 '18 at 6:06









          WW1WW1

          7,2751712




          7,2751712























              2












              $begingroup$

              You can use a change of base on the exponentials to see which should be larger:



              $$
              a^b = c^{blog_c(a)}
              $$



              We can use base 10 or, as I prefer, the natural log:



              $$
              3^{1000} = e^{1000ln(3)} approx e^{1099}
              $$



              and



              $$
              1000^{15} = e^{15ln(1000)} approx e^{104}
              $$



              It should be clear that the $5^text{th}$ root of $1000$ will be smaller, but you can proceed the same:



              $$
              sqrt[5]{1000} = 1000^frac{1}{5} = e^{frac{1}{5}ln(1000)} approx e^{1.4}
              $$



              EDIT:



              I just realized you were supposed to not use a calculator. Because of this, the base $10$ logarithm makes more sense:



              begin{align*}
              3^{1000} &= 10^{1000*log_{10}(3)} \
              1000^{15} &= 10^{15*log_{10}(1000)} = 10^{45} &text{$log_{10}left(10^3right) = 3$}\
              1000^{frac{1}{5}} &= 10^{frac{1}{5}log_{10}(1000)} = 10^{frac{3}{5}} &text{$log_{10}left(10^3right) = 3$}
              end{align*}



              All you need to do is show that $1000log_{10}(3) > 45$. We can approximate $log_{10}(3)$. We know that $3^2 < 10 < 3^3$, therefore $10^frac{1}{3} < 3 < 10^frac{1}{2}$ and thus $frac{1}{3} < log_{10}(3) < frac{1}{2}$. We use the smaller number (we want to "over" correct, by going as small as possible) and find that:



              $$
              frac{1000}{3} > 45
              $$



              Second Edit : $ln(1000)$



              Again, let's switch to base 10:



              $$
              log_a(x) = frac{log_b(x)}{log_b(a)}
              $$



              So we have:



              $$
              ln(1000) = frac{log_{10}(1000)}{log_{10}(e)} = frac{3}{log_{10}(e)}
              $$



              We can approximate $log_{10}(e)$ in the same way we approximated $log_{10}(3)$. We know that $2 < e < 3$. $2^3 < 10 < 2^4$ and (again) $3^2 < 10 < 3^3$, therefore $frac{1}{4} < log_{10}(2) < frac{1}{3}$ or $frac{1}{3} < log_{10}(3) < frac{1}{2}$. Be careful here, this will tell us that:



              begin{align*}
              frac{3}{frac{1}{2}} < ln(1000) < frac{3}{frac{1}{4}} \
              6 < ln(1000) < 12
              end{align*}



              If you can show that $10^frac{3}{5}$ is less than $6$ or greater than $12$, then you have your answer. So, surely $10^{frac{1}{5}} < 2$ (since $2^5 = 32$) but also $2^3= 8 > 6$ (so that's no good for our range). This is tricky to do, but can be done with some trial and error (hint: $1.5 = frac{3}{2}$ won't work--you'll end up proving that $frac{3}{2} < 10^{1/5}$).






              share|cite|improve this answer











              $endgroup$













              • $begingroup$
                Wait am I supposed to be able to estimate stuff like 1000*ln(3) without a calculator?
                $endgroup$
                – Will
                Dec 2 '18 at 5:47










              • $begingroup$
                @Will Yeah, I just realized that. I edited it. It should make more sense now.
                $endgroup$
                – Jared
                Dec 2 '18 at 5:56










              • $begingroup$
                I will read over it. I'm guessing it will make much more sense. Thank you so much!! One thing: Do you have any idea how to prove that ln 1000 is larger than the root but smaller than either the exponential or the power? You've proven three of the four numbers in the original problem, and I am so thankful. But do you have any thoughts on the other one, ln 1000?
                $endgroup$
                – Will
                Dec 2 '18 at 5:58


















              2












              $begingroup$

              You can use a change of base on the exponentials to see which should be larger:



              $$
              a^b = c^{blog_c(a)}
              $$



              We can use base 10 or, as I prefer, the natural log:



              $$
              3^{1000} = e^{1000ln(3)} approx e^{1099}
              $$



              and



              $$
              1000^{15} = e^{15ln(1000)} approx e^{104}
              $$



              It should be clear that the $5^text{th}$ root of $1000$ will be smaller, but you can proceed the same:



              $$
              sqrt[5]{1000} = 1000^frac{1}{5} = e^{frac{1}{5}ln(1000)} approx e^{1.4}
              $$



              EDIT:



              I just realized you were supposed to not use a calculator. Because of this, the base $10$ logarithm makes more sense:



              begin{align*}
              3^{1000} &= 10^{1000*log_{10}(3)} \
              1000^{15} &= 10^{15*log_{10}(1000)} = 10^{45} &text{$log_{10}left(10^3right) = 3$}\
              1000^{frac{1}{5}} &= 10^{frac{1}{5}log_{10}(1000)} = 10^{frac{3}{5}} &text{$log_{10}left(10^3right) = 3$}
              end{align*}



              All you need to do is show that $1000log_{10}(3) > 45$. We can approximate $log_{10}(3)$. We know that $3^2 < 10 < 3^3$, therefore $10^frac{1}{3} < 3 < 10^frac{1}{2}$ and thus $frac{1}{3} < log_{10}(3) < frac{1}{2}$. We use the smaller number (we want to "over" correct, by going as small as possible) and find that:



              $$
              frac{1000}{3} > 45
              $$



              Second Edit : $ln(1000)$



              Again, let's switch to base 10:



              $$
              log_a(x) = frac{log_b(x)}{log_b(a)}
              $$



              So we have:



              $$
              ln(1000) = frac{log_{10}(1000)}{log_{10}(e)} = frac{3}{log_{10}(e)}
              $$



              We can approximate $log_{10}(e)$ in the same way we approximated $log_{10}(3)$. We know that $2 < e < 3$. $2^3 < 10 < 2^4$ and (again) $3^2 < 10 < 3^3$, therefore $frac{1}{4} < log_{10}(2) < frac{1}{3}$ or $frac{1}{3} < log_{10}(3) < frac{1}{2}$. Be careful here, this will tell us that:



              begin{align*}
              frac{3}{frac{1}{2}} < ln(1000) < frac{3}{frac{1}{4}} \
              6 < ln(1000) < 12
              end{align*}



              If you can show that $10^frac{3}{5}$ is less than $6$ or greater than $12$, then you have your answer. So, surely $10^{frac{1}{5}} < 2$ (since $2^5 = 32$) but also $2^3= 8 > 6$ (so that's no good for our range). This is tricky to do, but can be done with some trial and error (hint: $1.5 = frac{3}{2}$ won't work--you'll end up proving that $frac{3}{2} < 10^{1/5}$).






              share|cite|improve this answer











              $endgroup$













              • $begingroup$
                Wait am I supposed to be able to estimate stuff like 1000*ln(3) without a calculator?
                $endgroup$
                – Will
                Dec 2 '18 at 5:47










              • $begingroup$
                @Will Yeah, I just realized that. I edited it. It should make more sense now.
                $endgroup$
                – Jared
                Dec 2 '18 at 5:56










              • $begingroup$
                I will read over it. I'm guessing it will make much more sense. Thank you so much!! One thing: Do you have any idea how to prove that ln 1000 is larger than the root but smaller than either the exponential or the power? You've proven three of the four numbers in the original problem, and I am so thankful. But do you have any thoughts on the other one, ln 1000?
                $endgroup$
                – Will
                Dec 2 '18 at 5:58
















              2












              2








              2





              $begingroup$

              You can use a change of base on the exponentials to see which should be larger:



              $$
              a^b = c^{blog_c(a)}
              $$



              We can use base 10 or, as I prefer, the natural log:



              $$
              3^{1000} = e^{1000ln(3)} approx e^{1099}
              $$



              and



              $$
              1000^{15} = e^{15ln(1000)} approx e^{104}
              $$



              It should be clear that the $5^text{th}$ root of $1000$ will be smaller, but you can proceed the same:



              $$
              sqrt[5]{1000} = 1000^frac{1}{5} = e^{frac{1}{5}ln(1000)} approx e^{1.4}
              $$



              EDIT:



              I just realized you were supposed to not use a calculator. Because of this, the base $10$ logarithm makes more sense:



              begin{align*}
              3^{1000} &= 10^{1000*log_{10}(3)} \
              1000^{15} &= 10^{15*log_{10}(1000)} = 10^{45} &text{$log_{10}left(10^3right) = 3$}\
              1000^{frac{1}{5}} &= 10^{frac{1}{5}log_{10}(1000)} = 10^{frac{3}{5}} &text{$log_{10}left(10^3right) = 3$}
              end{align*}



              All you need to do is show that $1000log_{10}(3) > 45$. We can approximate $log_{10}(3)$. We know that $3^2 < 10 < 3^3$, therefore $10^frac{1}{3} < 3 < 10^frac{1}{2}$ and thus $frac{1}{3} < log_{10}(3) < frac{1}{2}$. We use the smaller number (we want to "over" correct, by going as small as possible) and find that:



              $$
              frac{1000}{3} > 45
              $$



              Second Edit : $ln(1000)$



              Again, let's switch to base 10:



              $$
              log_a(x) = frac{log_b(x)}{log_b(a)}
              $$



              So we have:



              $$
              ln(1000) = frac{log_{10}(1000)}{log_{10}(e)} = frac{3}{log_{10}(e)}
              $$



              We can approximate $log_{10}(e)$ in the same way we approximated $log_{10}(3)$. We know that $2 < e < 3$. $2^3 < 10 < 2^4$ and (again) $3^2 < 10 < 3^3$, therefore $frac{1}{4} < log_{10}(2) < frac{1}{3}$ or $frac{1}{3} < log_{10}(3) < frac{1}{2}$. Be careful here, this will tell us that:



              begin{align*}
              frac{3}{frac{1}{2}} < ln(1000) < frac{3}{frac{1}{4}} \
              6 < ln(1000) < 12
              end{align*}



              If you can show that $10^frac{3}{5}$ is less than $6$ or greater than $12$, then you have your answer. So, surely $10^{frac{1}{5}} < 2$ (since $2^5 = 32$) but also $2^3= 8 > 6$ (so that's no good for our range). This is tricky to do, but can be done with some trial and error (hint: $1.5 = frac{3}{2}$ won't work--you'll end up proving that $frac{3}{2} < 10^{1/5}$).






              share|cite|improve this answer











              $endgroup$



              You can use a change of base on the exponentials to see which should be larger:



              $$
              a^b = c^{blog_c(a)}
              $$



              We can use base 10 or, as I prefer, the natural log:



              $$
              3^{1000} = e^{1000ln(3)} approx e^{1099}
              $$



              and



              $$
              1000^{15} = e^{15ln(1000)} approx e^{104}
              $$



              It should be clear that the $5^text{th}$ root of $1000$ will be smaller, but you can proceed the same:



              $$
              sqrt[5]{1000} = 1000^frac{1}{5} = e^{frac{1}{5}ln(1000)} approx e^{1.4}
              $$



              EDIT:



              I just realized you were supposed to not use a calculator. Because of this, the base $10$ logarithm makes more sense:



              begin{align*}
              3^{1000} &= 10^{1000*log_{10}(3)} \
              1000^{15} &= 10^{15*log_{10}(1000)} = 10^{45} &text{$log_{10}left(10^3right) = 3$}\
              1000^{frac{1}{5}} &= 10^{frac{1}{5}log_{10}(1000)} = 10^{frac{3}{5}} &text{$log_{10}left(10^3right) = 3$}
              end{align*}



              All you need to do is show that $1000log_{10}(3) > 45$. We can approximate $log_{10}(3)$. We know that $3^2 < 10 < 3^3$, therefore $10^frac{1}{3} < 3 < 10^frac{1}{2}$ and thus $frac{1}{3} < log_{10}(3) < frac{1}{2}$. We use the smaller number (we want to "over" correct, by going as small as possible) and find that:



              $$
              frac{1000}{3} > 45
              $$



              Second Edit : $ln(1000)$



              Again, let's switch to base 10:



              $$
              log_a(x) = frac{log_b(x)}{log_b(a)}
              $$



              So we have:



              $$
              ln(1000) = frac{log_{10}(1000)}{log_{10}(e)} = frac{3}{log_{10}(e)}
              $$



              We can approximate $log_{10}(e)$ in the same way we approximated $log_{10}(3)$. We know that $2 < e < 3$. $2^3 < 10 < 2^4$ and (again) $3^2 < 10 < 3^3$, therefore $frac{1}{4} < log_{10}(2) < frac{1}{3}$ or $frac{1}{3} < log_{10}(3) < frac{1}{2}$. Be careful here, this will tell us that:



              begin{align*}
              frac{3}{frac{1}{2}} < ln(1000) < frac{3}{frac{1}{4}} \
              6 < ln(1000) < 12
              end{align*}



              If you can show that $10^frac{3}{5}$ is less than $6$ or greater than $12$, then you have your answer. So, surely $10^{frac{1}{5}} < 2$ (since $2^5 = 32$) but also $2^3= 8 > 6$ (so that's no good for our range). This is tricky to do, but can be done with some trial and error (hint: $1.5 = frac{3}{2}$ won't work--you'll end up proving that $frac{3}{2} < 10^{1/5}$).







              share|cite|improve this answer














              share|cite|improve this answer



              share|cite|improve this answer








              edited Dec 2 '18 at 6:23

























              answered Dec 2 '18 at 5:43









              JaredJared

              5,20311116




              5,20311116












              • $begingroup$
                Wait am I supposed to be able to estimate stuff like 1000*ln(3) without a calculator?
                $endgroup$
                – Will
                Dec 2 '18 at 5:47










              • $begingroup$
                @Will Yeah, I just realized that. I edited it. It should make more sense now.
                $endgroup$
                – Jared
                Dec 2 '18 at 5:56










              • $begingroup$
                I will read over it. I'm guessing it will make much more sense. Thank you so much!! One thing: Do you have any idea how to prove that ln 1000 is larger than the root but smaller than either the exponential or the power? You've proven three of the four numbers in the original problem, and I am so thankful. But do you have any thoughts on the other one, ln 1000?
                $endgroup$
                – Will
                Dec 2 '18 at 5:58




















              • $begingroup$
                Wait am I supposed to be able to estimate stuff like 1000*ln(3) without a calculator?
                $endgroup$
                – Will
                Dec 2 '18 at 5:47










              • $begingroup$
                @Will Yeah, I just realized that. I edited it. It should make more sense now.
                $endgroup$
                – Jared
                Dec 2 '18 at 5:56










              • $begingroup$
                I will read over it. I'm guessing it will make much more sense. Thank you so much!! One thing: Do you have any idea how to prove that ln 1000 is larger than the root but smaller than either the exponential or the power? You've proven three of the four numbers in the original problem, and I am so thankful. But do you have any thoughts on the other one, ln 1000?
                $endgroup$
                – Will
                Dec 2 '18 at 5:58


















              $begingroup$
              Wait am I supposed to be able to estimate stuff like 1000*ln(3) without a calculator?
              $endgroup$
              – Will
              Dec 2 '18 at 5:47




              $begingroup$
              Wait am I supposed to be able to estimate stuff like 1000*ln(3) without a calculator?
              $endgroup$
              – Will
              Dec 2 '18 at 5:47












              $begingroup$
              @Will Yeah, I just realized that. I edited it. It should make more sense now.
              $endgroup$
              – Jared
              Dec 2 '18 at 5:56




              $begingroup$
              @Will Yeah, I just realized that. I edited it. It should make more sense now.
              $endgroup$
              – Jared
              Dec 2 '18 at 5:56












              $begingroup$
              I will read over it. I'm guessing it will make much more sense. Thank you so much!! One thing: Do you have any idea how to prove that ln 1000 is larger than the root but smaller than either the exponential or the power? You've proven three of the four numbers in the original problem, and I am so thankful. But do you have any thoughts on the other one, ln 1000?
              $endgroup$
              – Will
              Dec 2 '18 at 5:58






              $begingroup$
              I will read over it. I'm guessing it will make much more sense. Thank you so much!! One thing: Do you have any idea how to prove that ln 1000 is larger than the root but smaller than either the exponential or the power? You've proven three of the four numbers in the original problem, and I am so thankful. But do you have any thoughts on the other one, ln 1000?
              $endgroup$
              – Will
              Dec 2 '18 at 5:58













              1












              $begingroup$

              From $$e^2<3^2<10<e^3<3^3$$ we have $$2<2ln 3<ln 10<3<3ln 3 tag 1$$




              • Realize that $sqrt[5]{1000}<ln1000,$ as their fifth powers are in this order: $$1000<2^{10}<6^5<(ln1000)^5$$


              • Now, compute logarithmes of $ln 1000,;1000^{15},;3^{1000}$ and find their estimates with the use of $(1):$



              $$ln3<ln6< color{blue}{ln(ln 1000)}<ln9=2ln3 $$
              $$color{blue}{ln3^{1000}}=1000ln3$$
              $$90ln3<color{blue}{ln 1000^{15}}<135ln3$$
              The numbers are in the same order as their natural logarithmes.



              Putting together both parts, we conclude $$sqrt[5]{1000}<ln 1000<1000^{15}<3^{1000}.$$






              share|cite|improve this answer











              $endgroup$


















                1












                $begingroup$

                From $$e^2<3^2<10<e^3<3^3$$ we have $$2<2ln 3<ln 10<3<3ln 3 tag 1$$




                • Realize that $sqrt[5]{1000}<ln1000,$ as their fifth powers are in this order: $$1000<2^{10}<6^5<(ln1000)^5$$


                • Now, compute logarithmes of $ln 1000,;1000^{15},;3^{1000}$ and find their estimates with the use of $(1):$



                $$ln3<ln6< color{blue}{ln(ln 1000)}<ln9=2ln3 $$
                $$color{blue}{ln3^{1000}}=1000ln3$$
                $$90ln3<color{blue}{ln 1000^{15}}<135ln3$$
                The numbers are in the same order as their natural logarithmes.



                Putting together both parts, we conclude $$sqrt[5]{1000}<ln 1000<1000^{15}<3^{1000}.$$






                share|cite|improve this answer











                $endgroup$
















                  1












                  1








                  1





                  $begingroup$

                  From $$e^2<3^2<10<e^3<3^3$$ we have $$2<2ln 3<ln 10<3<3ln 3 tag 1$$




                  • Realize that $sqrt[5]{1000}<ln1000,$ as their fifth powers are in this order: $$1000<2^{10}<6^5<(ln1000)^5$$


                  • Now, compute logarithmes of $ln 1000,;1000^{15},;3^{1000}$ and find their estimates with the use of $(1):$



                  $$ln3<ln6< color{blue}{ln(ln 1000)}<ln9=2ln3 $$
                  $$color{blue}{ln3^{1000}}=1000ln3$$
                  $$90ln3<color{blue}{ln 1000^{15}}<135ln3$$
                  The numbers are in the same order as their natural logarithmes.



                  Putting together both parts, we conclude $$sqrt[5]{1000}<ln 1000<1000^{15}<3^{1000}.$$






                  share|cite|improve this answer











                  $endgroup$



                  From $$e^2<3^2<10<e^3<3^3$$ we have $$2<2ln 3<ln 10<3<3ln 3 tag 1$$




                  • Realize that $sqrt[5]{1000}<ln1000,$ as their fifth powers are in this order: $$1000<2^{10}<6^5<(ln1000)^5$$


                  • Now, compute logarithmes of $ln 1000,;1000^{15},;3^{1000}$ and find their estimates with the use of $(1):$



                  $$ln3<ln6< color{blue}{ln(ln 1000)}<ln9=2ln3 $$
                  $$color{blue}{ln3^{1000}}=1000ln3$$
                  $$90ln3<color{blue}{ln 1000^{15}}<135ln3$$
                  The numbers are in the same order as their natural logarithmes.



                  Putting together both parts, we conclude $$sqrt[5]{1000}<ln 1000<1000^{15}<3^{1000}.$$







                  share|cite|improve this answer














                  share|cite|improve this answer



                  share|cite|improve this answer








                  edited Dec 2 '18 at 18:47

























                  answered Dec 2 '18 at 11:52









                  user376343user376343

                  3,3282825




                  3,3282825






























                      draft saved

                      draft discarded




















































                      Thanks for contributing an answer to Mathematics Stack Exchange!


                      • Please be sure to answer the question. Provide details and share your research!

                      But avoid



                      • Asking for help, clarification, or responding to other answers.

                      • Making statements based on opinion; back them up with references or personal experience.


                      Use MathJax to format equations. MathJax reference.


                      To learn more, see our tips on writing great answers.




                      draft saved


                      draft discarded














                      StackExchange.ready(
                      function () {
                      StackExchange.openid.initPostLogin('.new-post-login', 'https%3a%2f%2fmath.stackexchange.com%2fquestions%2f3022284%2fcomparing-ln-1000-sqrt51000-31000-and-100015-without-calc%23new-answer', 'question_page');
                      }
                      );

                      Post as a guest















                      Required, but never shown





















































                      Required, but never shown














                      Required, but never shown












                      Required, but never shown







                      Required, but never shown

































                      Required, but never shown














                      Required, but never shown












                      Required, but never shown







                      Required, but never shown







                      Popular posts from this blog

                      Quarter-circle Tiles

                      build a pushdown automaton that recognizes the reverse language of a given pushdown automaton?

                      Mont Emei